Đến nội dung

Hình ảnh

Bất đẳng thức qua các kì thi toán quốc tế

* * * * - 3 Bình chọn imo olympiad kvant imo shortlist tst jbmo apmo mosp balkan

  • Please log in to reply
Chủ đề này có 64 trả lời

#41
dungxibo123

dungxibo123

    Sĩ quan

  • Thành viên
  • 330 Bài viết

Tiếp tục:

  • Bài 30: [Trần Nam Dũng, VMO 2008]

Cho x, y, z là các số thực không âm. thoả mãn xy + yz + xz khác 0. Chứng minh rằng:

$\sum \frac{1}{(x-y)^2}\geq \frac{4}{xy+yz+zx}$

không mất tổng quát : giả sử $ z=min\left\{x,y,z\right\}$

ta có  $\sum \frac{1}{(x-y)^{2}}-\frac{4}{xy+yz+zx}$

$=\frac {[(x-z)^2-3(x-z)(y-z)+(y-z)^2]^2}{(x - y)^2(y-z)^2(z-x)^2}$

$+\frac{4z(2x+2y-z)}{(x-z)(y-z)[3z^2+2z(x+y-2z)+(x-z)(y-z)]}\ge 0.$
em từng đọc bài này hồi năm 2015 nên nhớ :)


Bài viết đã được chỉnh sửa nội dung bởi dungxibo123: 26-11-2016 - 19:26

myfb : www.facebook.com/votiendung.0805
~~~~~~~~~~~~~~~~~~~~~~~~~~~~~~~~~~~~~o0o~~~~~~~~~~~~~~~~~~~~~~~~~~~~~~~~~~~~~~
SỢ HÃI giúp ta tồn tại

NGHỊ LỰC giúp ta đứng vững

KHÁT VỌNG giúp ta tiến về phía trước

Võ Tiến Dũng  

:like  :like  :like  :like  :like 

 

 


#42
dungxibo123

dungxibo123

    Sĩ quan

  • Thành viên
  • 330 Bài viết

Đóng góp:

Bài 33 (Azerbaijan JBMO TST 2015): cho $a,b,c \in \mathbb{R}^{+}$ thỏa $ a+b+c=1$

tìm chứng minh  $\sum \frac{7+2b}{1+a}\geq \frac{69}{4}$

p/s: em vẫn chưa giải ra ạ ~~ 


myfb : www.facebook.com/votiendung.0805
~~~~~~~~~~~~~~~~~~~~~~~~~~~~~~~~~~~~~o0o~~~~~~~~~~~~~~~~~~~~~~~~~~~~~~~~~~~~~~
SỢ HÃI giúp ta tồn tại

NGHỊ LỰC giúp ta đứng vững

KHÁT VỌNG giúp ta tiến về phía trước

Võ Tiến Dũng  

:like  :like  :like  :like  :like 

 

 


#43
Nguyenphuctang

Nguyenphuctang

    Sĩ quan

  • Banned
  • 499 Bài viết

Lời giải bài 33 : [Azerbaijan JBMO TST 2015]

 

$VT=2\left ( \sum \frac{1+a}{1+c} \right )+5\left ( \sum \frac{1}{1+a} \right )$

$\geq 2.3\sqrt{\frac{(1+a)(1+b)(1+c)}{(1+a)(1+b)(1+c)}}+\frac{5.9}{1+a+1+b+1+c}=\frac{69}{4}$



#44
Nguyenphuctang

Nguyenphuctang

    Sĩ quan

  • Banned
  • 499 Bài viết

Tiếp tục :

 

Bài 34: [APMO 2005]

 

Cho a, b, c > 0; abc =8 . Chứng minh rằng:

$\sum \frac{a^{2}}{\sqrt{(a^{3}+1)(b^{3}+1)}}\geq \frac{4}{3}$



#45
nguyen cong dat

nguyen cong dat

    Binh nhì

  • Thành viên mới
  • 15 Bài viết

không mất tổng quát : giả sử $ z=min\left\{x,y,z\right\}$

ta có  $\sum \frac{1}{(x-y)^{2}}-\frac{4}{xy+yz+zx}$

$=\frac {[(x-z)^2-3(x-z)(y-z)+(y-z)^2]^2}{(x - y)^2(y-z)^2(z-x)^2}$

$+\frac{4z(2x+2y-z)}{(x-z)(y-z)[3z^2+2z(x+y-2z)+(x-z)(y-z)]}\ge 0.$
em từng đọc bài này hồi năm 2015 nên nhớ :)

http://diendantoanho...cau-6-vmo-2008/ cac ban tham khao them



#46
dungxibo123

dungxibo123

    Sĩ quan

  • Thành viên
  • 330 Bài viết

Tiếp tục :

 

Bài 34: [APMO 2005]

 

Cho a, b, c > 0; abc =8 . Chứng minh rằng:

$\sum \frac{a^{2}}{\sqrt{(a^{3}+1)(b^{3}+1)}}\geq \frac{4}{3}$

có : $\frac{a^2 + 2}{2} = \frac{(a^2 - a + 1) + (a + 1)}{2} \geq \sqrt{(a^2 - a + 1)(a + 1)} = \sqrt{a^3 + 1}$ sau đó: bất đẳng thức cần chứng minh trở về 

 

$\sum \frac{a^{2}}{(a^{2}+2)(b^{2}+2)}\geq \frac{1}{3}$ ( bất đẳng thức dễ chứng minh khi quy đồng bung hết ra và dùng AM-GM ạ )

myfb : www.facebook.com/votiendung.0805
~~~~~~~~~~~~~~~~~~~~~~~~~~~~~~~~~~~~~o0o~~~~~~~~~~~~~~~~~~~~~~~~~~~~~~~~~~~~~~
SỢ HÃI giúp ta tồn tại

NGHỊ LỰC giúp ta đứng vững

KHÁT VỌNG giúp ta tiến về phía trước

Võ Tiến Dũng  

:like  :like  :like  :like  :like 

 

 


#47
Nguyenphuctang

Nguyenphuctang

    Sĩ quan

  • Banned
  • 499 Bài viết

Tiếp tục:

Bài 35: [Turkish TST 2007]

 

Cho a, b, c > 0; a + b + c = 1. Chứng minh rằng:

$\sum \frac{1}{ab++2c+2c^{2}}\geq \frac{1}{ab+bc+ca}$



#48
Nguyenphuctang

Nguyenphuctang

    Sĩ quan

  • Banned
  • 499 Bài viết

Lời giải bài 31 : [UKMO 2005]

 

Không mất tính tổng quát giả sử c = min{a,b,c}.

Áp dụng bất đẳng thức AM - GM ta có: 

$(a+b+c)\left ( \frac{1}{a}+\frac{1}{b}+\frac{1}{c} \right )=\frac{a+b+c}{b}\left ( \frac{b}{a}+1+\frac{b}{c} \right )\leq \frac{1}{4}\left ( \frac{a+b+c}{b}+\frac{b}{a}+\frac{b}{c}+1 \right )^{2}$

Ta cần phải chứng minh:

$2\left ( \frac{a}{b}+\frac{b}{c}+\frac{c}{a} \right )\geq \frac{a+b+c}{b}+\frac{b}{a}+\frac{b}{c}+1$

$\Leftrightarrow (a-c)\left ( \frac{1}{b}+\frac{b}{ac}-\frac{2}{a} \right )\geq 0$

Luôn đúng vì: 

$a-c\geq 0$ và $\frac{1}{b}+\frac{b}{ac}\geq \frac{2}{\sqrt{ac}}\geq \frac{2}{a}$

Đẳng thức xảy ra khi a = b = c.                                                                                                                                           $\square$



#49
nguyen cong dat

nguyen cong dat

    Binh nhì

  • Thành viên mới
  • 15 Bài viết

Lời giải bài 31 : [UKMO 2005]

 

Không mất tính tổng quát giả sử c = min{a,b,c}.

Áp dụng bất đẳng thức AM - GM ta có: 

$(a+b+c)\left ( \frac{1}{a}+\frac{1}{b}+\frac{1}{c} \right )=\frac{a+b+c}{b}\left ( \frac{b}{a}+1+\frac{b}{c} \right )\leq \frac{1}{4}\left ( \frac{a+b+c}{b}+\frac{b}{a}+\frac{b}{c}+1 \right )^{2}$

Ta cần phải chứng minh:

$2\left ( \frac{a}{b}+\frac{b}{c}+\frac{c}{a} \right )\geq \frac{a+b+c}{b}+\frac{b}{a}+\frac{b}{c}+1$

$\Leftrightarrow (a-c)\left ( \frac{1}{b}+\frac{b}{ac}-\frac{2}{a} \right )\geq 0$

Luôn đúng vì: 

$a-c\geq 0$ và $\frac{1}{b}+\frac{b}{ac}\geq \frac{2}{\sqrt{ac}}\geq \frac{2}{a}$

Đẳng thức xảy ra khi a = b = c.                                                                                                                                           $\square$

bạn ơi phương pháp này là phuong pháp gì vậy ,mình thấy nhiều trên diễn dàn mà ko biết sd



#50
Nguyenphuctang

Nguyenphuctang

    Sĩ quan

  • Banned
  • 499 Bài viết

Tiếp tục : 

 

Bài 36 : [Mongolia 2007]

 

Cho a, b, c > 0. Chứng minh rằng:

 

$\frac{a}{b}+\frac{b}{c}+\frac{c}{a}\geq 3\sqrt{\frac{a^{2}+b^{2}+c^{2}}{ab+bc+ca}}$



#51
dungxibo123

dungxibo123

    Sĩ quan

  • Thành viên
  • 330 Bài viết

Lời giải bài 31 : [UKMO 2005]

 

Không mất tính tổng quát giả sử c = min{a,b,c}.

Áp dụng bất đẳng thức AM - GM ta có: 

$(a+b+c)\left ( \frac{1}{a}+\frac{1}{b}+\frac{1}{c} \right )=\frac{a+b+c}{b}\left ( \frac{b}{a}+1+\frac{b}{c} \right )\leq \frac{1}{4}\left ( \frac{a+b+c}{b}+\frac{b}{a}+\frac{b}{c}+1 \right )^{2}$

Ta cần phải chứng minh:

$2\left ( \frac{a}{b}+\frac{b}{c}+\frac{c}{a} \right )\geq \frac{a+b+c}{b}+\frac{b}{a}+\frac{b}{c}+1$

$\Leftrightarrow (a-c)\left ( \frac{1}{b}+\frac{b}{ac}-\frac{2}{a} \right )\geq 0$

Luôn đúng vì: 

$a-c\geq 0$ và $\frac{1}{b}+\frac{b}{ac}\geq \frac{2}{\sqrt{ac}}\geq \frac{2}{a}$

Đẳng thức xảy ra khi a = b = c.                                                                                                                                           $\square$

có thể tham khảo cách làm trong sách của Thầy Võ Quốc Bá Cẩn ( Cauchy- Schwarz)


myfb : www.facebook.com/votiendung.0805
~~~~~~~~~~~~~~~~~~~~~~~~~~~~~~~~~~~~~o0o~~~~~~~~~~~~~~~~~~~~~~~~~~~~~~~~~~~~~~
SỢ HÃI giúp ta tồn tại

NGHỊ LỰC giúp ta đứng vững

KHÁT VỌNG giúp ta tiến về phía trước

Võ Tiến Dũng  

:like  :like  :like  :like  :like 

 

 


#52
Nguyenphuctang

Nguyenphuctang

    Sĩ quan

  • Banned
  • 499 Bài viết

1 cách giải khác cho bài 31: 
http://diendantoanho...-2005/?p=663282



#53
Nguyenphuctang

Nguyenphuctang

    Sĩ quan

  • Banned
  • 499 Bài viết

Lời giải bài 36:

https://www.facebook...57524429&type=3


Bài viết đã được chỉnh sửa nội dung bởi Nguyenphuctang: 28-11-2016 - 16:39


#54
Nguyenphuctang

Nguyenphuctang

    Sĩ quan

  • Banned
  • 499 Bài viết

Đáp án bài 27 :

http://diendantoanho...yzxy1/?p=663271



#55
Uchiha Sasuke 88

Uchiha Sasuke 88

    Binh nhì

  • Thành viên mới
  • 13 Bài viết

Câu 1 nếu khai triển ra là bđt Schur bậc 3 nên có thể giải được

$a^{2}b+a^{2}c+b^{2}c+b^{2}a+c^{2}a+c^{2}b\leq a^{3}+b^{3}+c^{3}+3abc$

$\Leftrightarrow a(a-b)(a-c)+b(b-a)(b-c)+c(c-a)(c-b)\geq 0$



#56
Uchiha Sasuke 88

Uchiha Sasuke 88

    Binh nhì

  • Thành viên mới
  • 13 Bài viết

Em xin đóng góp bài 12

Nếu  $x\geq 1$ thì $x^{4}\geq x$ suy ra Đpcm

Nếu x$\leq 1$ thì 

$x\leq \frac{1}{2} \Rightarrow VP\leq 0 \Rightarrow dpcm$

$x\geq \frac{1}{2}: x^{4}+\frac{1}{4}+\frac{1}{4}\geq x^{2}+\frac{1}{4}\geq x$


Bài viết đã được chỉnh sửa nội dung bởi JUV: 02-12-2016 - 17:41


#57
Mr Cooper

Mr Cooper

    Sĩ quan

  • Thành viên
  • 496 Bài viết

Bài 37:  Cho a,b,c thuộc đoạn [-2;3] và a+b+c=2. CMR:

 

\[{a^2} + {b^2} + {c^2} \le 14\]



#58
takarin1512

takarin1512

    Trung sĩ

  • Thành viên
  • 104 Bài viết

Bài 37:  Cho a,b,c thuộc đoạn [-2;3] và a+b+c=2. CMR:

 

\[{a^2} + {b^2} + {c^2} \le 14\]

Không mất tỉnh tổng quát, giả sử $-2\leq a\leq b\leq c\leq 3$.

Ta có $a^2+b^2+c^2=c\left ( c-b \right )+\left ( c+b \right )\left ( b-a \right )+\left ( a+b+c \right )a\leq 3\left ( c-b \right )+4\left ( b-a \right )+2a=3c+b-2a=2+2c-3a\leq 2+6-\left ( -6 \right )=14$

Đẳng thức xảy ra khi $\left ( a,b,c \right )=\left ( -2;1;3 \right )$ và các hoán vị của chúng.



#59
Nguyenphuctang

Nguyenphuctang

    Sĩ quan

  • Banned
  • 499 Bài viết

Bài 37:  Cho a,b,c thuộc đoạn [-2;3] và a+b+c=2. CMR:

 

\[{a^2} + {b^2} + {c^2} \le 14\]

Bạn có đọc kĩ nội quy của topic này chưa? Bài này có nguồn từ đâu? có phải trên các tạp chí hay các kì thi? 



#60
dungxibo123

dungxibo123

    Sĩ quan

  • Thành viên
  • 330 Bài viết

Bài 38 : ( Polish MO 2009) 

cho $n \geq 1$ và $ a,b,c>0$

chứng minh $\sum \frac{a^{n+1}}{b+c}\geq(\sum \frac{a^{n}}{b+c})\sqrt[n]{\frac{\sum a^{n}}{3}}$


myfb : www.facebook.com/votiendung.0805
~~~~~~~~~~~~~~~~~~~~~~~~~~~~~~~~~~~~~o0o~~~~~~~~~~~~~~~~~~~~~~~~~~~~~~~~~~~~~~
SỢ HÃI giúp ta tồn tại

NGHỊ LỰC giúp ta đứng vững

KHÁT VỌNG giúp ta tiến về phía trước

Võ Tiến Dũng  

:like  :like  :like  :like  :like 

 

 






Được gắn nhãn với một hoặc nhiều trong số những từ khóa sau: imo, olympiad, kvant, imo shortlist, tst, jbmo, apmo, mosp, balkan

0 người đang xem chủ đề

0 thành viên, 0 khách, 0 thành viên ẩn danh